LSAT and Law School Admissions Forum

Get expert LSAT preparation and law school admissions advice from PowerScore Test Preparation.

 Emily Haney-Caron
PowerScore Staff
  • PowerScore Staff
  • Posts: 577
  • Joined: Jan 12, 2012
|
#34999
Hi mankariousc,

A would work if we had any info about economic conditions improving, but we don't. So, although there is another possible cause that exists, there is no evidence that happened here, and so A doesn't weaken the argument.

Hope that helps!
 AnnBar
  • Posts: 33
  • Joined: Mar 24, 2017
|
#35226
adlindsey wrote:I actually looked at A as an alternate cause. I thought we could bring in new information with this question type?

I have the same question quoted above. I thought Weaken Qs allowed new information.

Thanks
 AthenaDalton
PowerScore Staff
  • PowerScore Staff
  • Posts: 296
  • Joined: May 02, 2017
|
#35263
AnnBar wrote:
adlindsey wrote:I actually looked at A as an alternate cause. I thought we could bring in new information with this question type?

I have the same question quoted above. I thought Weaken Qs allowed new information.

Thanks
Hi Ann,

We can bring in new types of information in these question types -- in fact, the correct answer, answer choice (E), does exactly that.

The problem with answer choice (A) is that it tells us that improving economic conditions decrease crime, but stops short of saying that economic conditions have improved in this particular city. Without knowing whether or not this city is faring well or poorly economically, we can't deduce anything about the impact on crime rates.

By contrast, answer choice (E) does give us concrete information about something that happened in this city that provides an alternate cause for the stated effect (falling crime rates).

I hope this helps!

Athena Dalton
 Chas
  • Posts: 3
  • Joined: Jul 31, 2017
|
#38945
I've read the explanations for this question but am still not clear on why answer c) is incorrect. While I see the reasoning for e) and I grant that it could be considered the most correct, the reasoning for striking c) seems unconvincing. First of all, the stimulus switches from 'violent crime rate' in the premise to 'crime rate' in the conclusion. I rephrased an answer that would have 'crime rate' in it. If the overall crime rate had been going down then it opens the door to the idea that there could be many other reasons crime was going down in general. This seems particularly attractive since the premise makes no connection between the harsher sentencing and the crime rate in general.

Also the idea that b) is wrong because judges had for many years been imposing harsh sentences is wrong because they'd basically been doing what the new sentencing law forces them to do for many years and there hadn't been a drop in the crime rate assumes that there hadn't already been a drop in crime in previous years as well. Since we have no idea if there had or hadn't and we don't know how many 'many' years mean we have no way to apply your logic. It is as open ended a possibility as e) since it started 2 years prior. What is the substantive difference between 3 years of harsher sentencing and 2 years of more cops?
 Adam Tyson
PowerScore Staff
  • PowerScore Staff
  • Posts: 5400
  • Joined: Apr 14, 2011
|
#38996
I'll see if I can shed a little more light on it for you, Chas, but your analysis of answer B is actually a good reason why we should reject it. It's too vague! As you said, "we have no idea if there had or hadn't and we don't know how many 'many' years mean." So what does answer B tell us? That for some undetermined amount of time prior to the new policy being enacted, judges had enacted unusually harsh penalties for some crimes. Violent crimes? We don't know. As harsh as the new policy requires? We don't know. Does answer B weaken the causal claim, that the new policy is what caused the decline that happened only after it was enacted, by giving us any kind of alternate cause, cause without effect, effect without cause, reversed cause and effect, or data problem? It does not. The policy was enacted and then crime dropped, and no other policy was enacted in that time. B just leaves us scratching our head and wondering whether it matters or not. If you are seeing it as a "cause was present but effect was not", think again, because harsh sentencing is NOT the same cause as a new policy of mandatory sentencing. Similar, perhaps, if it was applied to the exact same crimes and the sentences were just as harsh as the new mandatory sentences, but that's way too much unknown info that we have to imagine in order to help this answer. Don't help the answers, but take them as they are.

Answer C also doesn't hurt the causal claim. If violent crime is down 15 percent, and overall crime is down 5 percent, that could mean any number of things. Maybe nonviolent crime increased? Maybe violent crime has always made up a relatively small percentage of total crime, and so it's reduction had a disproportionately low impact on the total? We don't know what these varying percentages tell us, but they definitely do NOT suggest that there was some other cause, or that the cause was present without the effect, for example. Remember, the policy enacted applied only to violent crimes, so changes in the rate of nonviolent crime, if any, are outside the scope of the causal argument here. Maybe the harsher penalties for violent crimes caused some criminals to switch from carjacking and mugging to identity theft and embezzlement? Maybe, but the new policy could still be the cause of the decrease in the rate of violent crime.

Answer E tells us that even though no other new policies were enacted last year, there was a policy enacted in a prior year that may have had an impact on the crime rate. We can't know for sure that more police causes less crime, but this at least gets us around the author's premise about no new policies. If that new policy, enacted prior to last year, might have caused some reduction in crime (either by getting the criminals off the streets, or by deterring crime due to a higher police presence, or just by causing criminals to change their minds about crime because they know the chances of getting caught are higher), then that's good enough. Answer E introduces an element of doubt into the argument, because it directly undermines the weight of one key premise (that "no other major policy changes were made last year"). Even if we don't accept the increase in the number of police as an alternate cause, it weakens the argument just by devaluing that premise! That's all you need in a good weaken answer - just an element of doubt. E gives us that without any outside help or creative imagination required, simply because of the way it undermines that crucial premise.

Keep at it!
 lunsandy
  • Posts: 61
  • Joined: Oct 14, 2017
|
#41521
Hi Powerscore,

Upon reviewing, I can see why E gives an alternative factor, as well as effect without the cause (drop in crime rate without the major policy changes in place yet). However, I am struggling to understand why it is correct that the stim talks about crime rate in % and answer E is comparing something in number (#), new police offers each year. I thought when the stim is talking about % we need another answer that matches with the stim (%). Thats why I got rid of E because of the mismatch.

Also, the conclusion talks about "crime rate" in general whereas, the stim talks about "violent crime rate." So is the conclusion saying because there is an increase in violent crime rate and put in prison there is a decrease in crime rate in general? Or the conclusion is equating crime rate = violent crime rate as being the same thing?

Thanks a lot!
 Claire Horan
PowerScore Staff
  • PowerScore Staff
  • Posts: 408
  • Joined: Apr 18, 2016
|
#42110
Hi Lunsandy,

Thanks for the specificity of your analysis! Because there is a lot going on in your question, I will go line by line in my response:
Upon reviewing, I can see why E gives an alternative factor, as well as effect without the cause (drop in crime rate without the major policy changes in place yet).
(E) does give an alternate cause, the hiring of more police officers, but this is not an example of "effect without the cause." The stimulus does not say there was a drop in the violent crime rate before the mandatory sentencing law was enacted, and you can't assume there was. The stimulus gives up no information about whether violent crime increased, decreased, or stayed the same in the year before the mandatory sentencing law was enacted.

Your next question refers to the numbers and percents in the stimulus:
However, I am struggling to understand why it is correct that the stim talks about crime rate in % and answer E is comparing something in number (#), new police offers each year. I thought when the stim is talking about % we need another answer that matches with the stim (%). Thats why I got rid of E because of the mismatch.
The flaw in your reasoning is that you are generalizing a little too much about Powerscore's number/percentage tips. The tip is that if you are only told a raw #, you do not know anything about a %, and if you are only told a %, you don't know anything about a number. So you can't simply see a % in the stimulus and a # in the answer choice and eliminate the answer based on that. The other inaccuracy in your statement is that the tip about %s and #s applies when you are talking about the same quantity. For example, knowing the percentage of women in a law school class does not tell you the number of women in the same class. You would have to know the total number of students in the class to determine the % from the # or vice-versa. But in this problem, the % is about violent crimes (crime rate means # of crimes divided by # of people). But the # is the number of police officers hired. It's apples and oranges.

Your last question is about the shift in language from "violent crime rate" to "crime rate":
Also, the conclusion talks about "crime rate" in general whereas, the stim talks about "violent crime rate." So is the conclusion saying because there is an increase in violent crime rate and put in prison there is a decrease in crime rate in general? Or the conclusion is equating crime rate = violent crime rate as being the same thing?
Let's look closely, though. The conclusion says "the drop in the crime rate," which refers to the drop previously mentioned, the drop in the violent crime rate.

Compare to this example: Yesterday I saw a black cat. The cat...

I am less specific in my second sentence, but it is clear I am referring to the black cat because I didn't mention any others.

I hope this helped. Great questions!
 hope
  • Posts: 84
  • Joined: Feb 13, 2018
|
#68450
I have a question about the first question in the Webinar/9/3/19 Logical Reasoning Causality Secrets.
This is a Weaken question. The answer ended up being E. But I thought that it could also be A. Why isn't it A? To me, the economic conditions that decrease crime can also be an alternate cause to the reduction in crime, which would weaken the argument.

Can you break down for me every answer to Question 1 and explain why it is not the correct answer and why according to the 5 points that we are supposed to be looking for: (1) alternate cause (2) cause but no effect (3) effect but no cause (4) reversal (5) statistical flaw with data. :0
User avatar
 Dave Killoran
PowerScore Staff
  • PowerScore Staff
  • Posts: 5978
  • Joined: Mar 25, 2011
|
#68459
Hi Hope,

Thanks for the question! That's the Police Commissioner, right? There's a complete explanation here for that question: https://forum.powerscore.com/lsat/viewtopic.php?t=9219.

Please let us know if that helps. Thanks!

[admin note: we moved the original question and its responses to the correct thread, also linked above]
 hope
  • Posts: 84
  • Joined: Feb 13, 2018
|
#68544
The reason why I thought that A could be the answer because it presented itself as an alternate cause against the new policy. And the reason why I thought that it was a suitable answer is because since Weaken is in family 3, like family 2 it can have new information that is not seen in the stimulus. The explanation that you sent me seems to suggest that the new information of economic conditions could not be correct because it presented new info not found in the stimulus. Which is it? Can the correct answer for a weaken question present new info, especially since it is in family 3? Please let me know. Thank you.

Get the most out of your LSAT Prep Plus subscription.

Analyze and track your performance with our Testing and Analytics Package.